Você está na página 1de 46

3.1.

Visualize:

Solve: (a) To find A + B , we place the tail of vector B on the tip of vector A and connect the tail of vector A with the tip of vector B. (b) Since A B = A + ( B) , we place the tail of the vector ( B ) on the tip of vector A and then connect the tail

of vector A with the tip of vector ( B ) .

3.2.

Visualize:

Solve: (a) To find A + B , we place the tail of vector B on the tip of vector A and then connect vector As tail with vector Bs tip. (b) To find A B , we note that A B = A + ( B) . We place the tail of vector B on the tip of vector A and

then connect vector As tail with the tip of vector B.

3.3.

Visualize:

Solve:

Vector E points to the left and up, so the components Ex and E y are negative and positive,

respectively, according to the Tactics Box 3.1. (a) Ex = E cos (b) Ex = E sin

and

E y = E sin .

and

E y = E cos .

Assess: Note that the role of sine and cosine are reversed because we are using a different angle. and are complementary angles.

3.4.

Visualize: The position vector r whose magnitude r is 10 m has an x-component of 6 m. It makes an angle with the + x -axis in the first quadrant.

Solve:

Using trigonometry, rx = r cos , or 6 m = (10 m)cos . This gives = 53.1. Thus the y-component of

the position vector r is ry = r sin = (10 m)sin 53.1 = 8 m.


Assess: The y-component is positive since the position vector is in the first quadrant.

3.5.

Visualize:

The figure shows the components vx and vy, and the angle .

Solve:

We have, v y = v sin 40, or 10 m/s = v sin 40, or v = 15.56 m/s.

Thus the x-component is vx = v cos 40 = (15.56 m/s ) cos 40 = 12 m/s. Assess: The x-component is positive since the position vector is in the fourth quadrant.

3.6.

Visualize:

We will follow rules in Tactics Box 3.1. Solve: (a) Vector r points to the right and down, so the components rx and ry are positive and negative, respectively:

rx = r cos = (100 m)cos 45 = 70.7 m

ry = r sin = (100 m)sin 45 = 70.7 m

(b) Vector v points to the right and up, so the components vx and v y are both positive:

vx = v cos = (300 m /s) cos 20 = 282 m/s


(c) Vector a has the following components:

v y = v sin = (300 m/s)sin 20 = 103 m/s

ax = a cos = (5.0 m/s 2 )cos90 = 0 m/s 2

a y = a sin = (5.0 m/s 2 )sin 90 = 5.0 m/s 2

Assess: The components have same units as the vectors. Note the minus signs we have manually inserted according to Tactics Box 3.1.

3.7.

Visualize:

We will follow the rules given in Tactics Box 3.1. Solve: v y = (5 cm/s)cos90 = 0 cm/s (a) vx = (5 cm/s)sin 90 = 5 cm/s (b) ax = (10 m/s 2 )sin 40 = 6.4 m/s 2 (c) Fx = (50 N)sin 36.9 = 30 N
a y = (10 m/s 2 )cos 40 = 7.7 m/s 2

Fy = (50 N)cos36.9 = 40 N

Assess: The components have the same units as the vectors. Note the minus signs we have manually inserted according to Tactics Box 3.1.

3.8.

Visualize:

The components of the vector C and D, and the angles are shown. Solve:

For C we have C x = (3.15 m)cos15 = 3.04 m and C y = (3.15 m)sin15 = 0.815 m. For D we

have Dx = 25.6sin 30 = 12.8 and Dy = 25.67cos30 = 22.2.

Assess: The components of the vector C have the same units as C itself. Dx and Dy are unitless because D is without units. Note the minus signs we have manually inserted following rules of Tactics Box 3.1.

3.9.

Visualize:

Solve: The magnitude of the vector is E = ( Ex ) 2 + ( E y ) 2 = (125 V/m) 2 + ( 250 V/m) 2 = 280 V/m. In the j expression for E , the and +i means that E is in quadrant IV. The angle is below the positive x-axis. We have:

= tan 1
Assess:

| Ey | Ex

250 V/m 1 = tan 1 = tan 2 = 63.4 125 V/m

Since | E y | > | Ex | , the angle made with the +x-axis is larger than 45. = 45 for | E y | = | Ex | .

3.10.

Visualize:

Solve:

(a) Using the formulas for the magnitude and direction of a vector, we have:

B = ( 4) 2 + (4) 2 = 5.7
(b) r = (2 cm) 2 + (1 cm) 2 = 2.2 cm
1 2

= tan 1 = tan 1 1 = 45

4 4

= tan 1 = tan 1 0.5 = 26.6

100 = tan 1 10 = 84.3 10 10 (d) a = (10 m/s 2 ) 2 + (20 m/s 2 ) 2 = 22.4 m/s 2 = tan 1 = tan 1 0.5 = 26.6 20 Assess: Note that 45 when | E y | | Ex |, where is the angle made with the x-axis. On the other hand,
(c) v = (10 m/s) 2 + (100 m/s) 2 = 100.5 m/s

= tan 1

> 45 when | E y | > | Ex | .

3.11.

Visualize:

Solve:

(a) Using the formulas for the magnitude and direction of a vector, we have:

A = (4) 2 + ( 6) 2 = 7.21
(b) r = (50 m) 2 + (80 m) 2 = 94.3 m

= tan 1 = tan 1 1.5 = 56.3


ry 1 80 m = tan = 58.0 rx 50 m

6 4

= tan 1

40 = tan 1 2 = 63.4 20 2 (d) a = (2 m/s 2 ) 2 + (6 m/s 2 ) 2 = 6.3 m/s 2 = tan 1 = tan 1 0.33 = 18.4 6 Assess: Note that the angle made with the x-axis is smaller than 45 whenever | E y | < | Ex | , = 45 for

(c) v = (20 m/s) 2 + (40 m/s) 2 = 44.7 m/s

= tan 1

| E y | = | Ex |, and > 45 for | E y | > | Ex | . In part (d), is with the y-axis, where the opposite of this rule applies.

3.12.

Visualize:

We have C = A B or C = A + ( B), where B = ( B, direction opposite B ). Look back at Tactics Box 1.2, which shows how to perform vector subtraction graphically. Solve: To obtain vector C from A and B, we place the tail of B on the tip of A, and then use the tip-totail rule of graphical addition.

3.13.

Visualize:

The vectors A, B, and C = A + B are shown.

Solve:

(a) We have A = 5i + 2 and B = 3i 5 . Thus, C = A + B = (5i + 2 ) + (3i 5 ) = 2i 3 . j j j j j

(b) Vectors A, B, and C are shown with their tails together. (c) Since C = 2i 3 = C xi + C y , Cx = 2, and C y = 3. Therefore, the magnitude and direction of C are j j

C = (2) 2 + (3) 2 = 3.6

= tan 1

| Cy |

3 = tan 1 = 56 below the + x-axis Cx 2

Assess: The vector C is to the right and down, thus implying a negative y-component and positive xcomponent, as obtained above. Also > 45 since | C y | > | Cx | .

3.14.

Visualize:

Solve:

(a) We have A = 5i + 2 , B = 3i 5 , and B = +3i + 5 . Thus, D = A + ( B) = 8i + 7 . j j j j

(b) Vectors A, B and D are shown in the above figure. (c) Since D = 8i + 7 = Dxi + Dy , Dx = 8 and Dy = 7. Therefore, the magnitude and direction of D are j j

D = (8) 2 + (7) 2 = 10.6


Assess:

= tan 1

Dy 1 7 = tan = 41 8 Dx

Since | Dy | < | Dx | , the angle is less than 45, as it should be.

3.15.

Visualize:

Solve:

(a) We have A = 5i + 2 and B = 3i 5 . This means 2 A = 10i + 4 and 3B = 9i 15 . Hence, j j j j E = 2 A + 3B = 1i 11 . j

(b) Vectors A , B, and E are shown in the above figure. (c) From the E vector, Ex = 1 and E y = 11 . Therefore, the magnitude and direction of E are
E = (1) 2 + (11) 2 = 11.05

= tan 1

Ex 1 = tan 1 = 5.19 right of the y -axis |E | 11 y

Assess: Note that is the angle made with the y-axis, and that is why = tan 1 ( Ex /| E y |) rather than

tan 1 (| E y |/ Ex ), which would be the case if were the angle made with the x-axis.

3.16.

Visualize:

Solve:

We have A = 5i + 2 and B = 3i 5 . j j + 22 = Fxi + Fy with Fx = 17 and Fy = 22. F = A 4 B = 17i j j

(a)

This

means

4 B = +12i + 20 . j

Hence,

(b) The vectors A, B, and F are shown in the above figure. (c) The magnitude and direction of F are

F = Fx2 + Fy2 = (17) 2 + (22) 2 = 27.8

= tan 1
Assess:

Fy 1 22 = tan = 52.3 17 Fx

Fy > Fx implies > 45, as is observed.

3.17.

Solve: A different coordinate system can only mean a different orientation of the grid and a different origin of the grid. (a) False, because the size of a vector is fixed. (b) False, because the direction of a vector in space is independent of any coordinate system. (c) True, because the orientation of the vector relative to the axes can be different.

3.18.

Visualize:

Solve:

j In coordinate system I, A = (4 m) , so Ax = 0 m and Ay = 4 m. The vector B makes an angle of

60 counterclockwise from vertical, which makes it have an angle of = 30 with the x-axis. Since B points to the left and up, it has a negative x-component and a positive y-component. That is, Bx = (5.0 m)cos30 = 4.3 m and By = + (5.0 m)sin 30 = 2.5 m. Thus, B = (4.3 m)i + (2.5 m) . j
In coordinate system II, A points to the left and down, and makes an angle of 30 with the y-axis. Therefore, Ax = (4.0 m)sin 30 = 2.0 m and Ay = (4.0)cos30 = 3.5 m. This implies A = (2.0 m)i (3.5 m) . j The vector
B makes an angle of 30 with the +y-axis and is to the left and up. This means we have to manually insert a x-component. Bx = B sin 30 = (5.0 m)sin 30 = 2.5 m, and minus sign with the + (4.3 m) . j By = + B cos30 = (5.0 m)cos30 = 4.3 m. Thus B = (2.5 m)i

3.19. Visualize: Refer to Figure EX3.19. The velocity vector v points west and makes an angle of 30 with the x-axis. v points to the left and up, implying that vx is negative and v y is positive.
Solve: Assess: We have

vx = v cos30

= (100 m/s)cos30

= 86.6 m/s

and

v y = + v sin 30 = (100 m/s)sin 30 = 50.0 m/s. vx and v y have the same units as v .

3.20.

Visualize:

(a)

Solve: (b) The components of the vectors A, B, and C are Ax = ( 3.0 m ) cos 20 = 2.8 m and Ay = ( 3.0 m ) sin 20= 1.0 m ; Bx = 0 m and By = 2 m;

Cx = ( 5.0 m ) cos70 = 1.71 m and C y = ( 5.0 m ) sin 70= 4.7 m. This means the vectors can be written, A = (2.8 m)i (1.0 m) j B = (2.0 m) j C = (1.71 m)i (4.7 m) j

(c) We have D = A + B + C = (1.09 m)i (3.7 m) . This means j


D = (1.09 m) 2 + (3.7 m) 2 = 3.9 m

= tan 1

3.9 = tan 1 3.58 = 74 1.09

The direction of D is south of east, 7 4 b elow the positive x-axis.

3.21.

Visualize:

Solve: Using the method of tail-to-tip graphical addition, the diagram shows the resultant for D + E + F in (a), the resultant for D + 2 E in (b), and the resultant for D 2 E + F in (c).

3.22.

Solve: We

have

E = Exi + E y = 2i + 3 , j j

which

means

Ex = 2

and

E y = 3.

Also,

j F = Fxi + Fy = 2i 2 , which means Fx = 2 and Fy = 2. j


2 (a) The magnitude of E is given by E = Ex2 + E y = (2) 2 + (3) 2 = 3.6 and the magnitude of F is given by

F = Fx2 + Fy2 = (2) 2 + (2) 2 = 2.8.

j (b) Since E + F = 4i + 1 , the magnitude of E + F is

(4) 2 + (1) 2 = 4.1.

j (c) Since E 2 F = (2i + 3 ) 2(2i 2 ) = 6i + 1 , the magnitude of E 2 F is j j

(6) 2 + (1) 2 = 6.1.

Solve: We have r = (5i + 4 )t 2 m. This means that r does not change the ratio of its components as t j increases, that is, the direction of r is constant. The magnitude of r is given by

3.23.

r = (5t 2 ) 2 + (4t 2 ) 2 m = (6.40t 2 ) m. (a) The particles distance from the origin at t = 0 s, t = 2 s, and t = 5 s is 0 m, 25.6 m, and 160 m. dr dt 2 (b) The particles velocity is v = = (5i + 4 ) j m/s = (5i + 4 )2t m/s = (10i + 8 )t m/s j j dt dt
(c) The magnitude of the particles velocity is given by v = (10 t ) 2 + (8t ) 2 = 12.8t m/s. The particles speed at t = 0 s, t = 2 s, and t = 5 s is 0 m/s, 25.6 m/s , and 64.0 m/s .

3.24.

Visualize:

Solve: (a) Vector C is the sum of vectors A and B, which is obtained using the tip-to-tip rule of graphical addition. Its magnitude is measured to be 4.7 and its angle made with the +x-axis is measured to be 33. (b) Using the law of cosines, C 2 = A2 + B 2 2 AB cos , and the geometry of parallelograms, which shows that = 180 ( B A ) = 180 (60 20) = 140, we obtain C = (3) 2 + (2) 2 2(3)(2)cos(140) = 4.71

Using the law of sines:


sin sin140 = 15.8 = 2 4.71 Thus, C = + 20 = 35.8. (c) We have:

Ax = A cos A = 3cos 20 = 2.82 Bx = B cos B = 2cos60 = 1.00

Ay = A sin A = 3sin 20 = 1.03 By = B sin B = 2sin 60 = 1.73

This means: Cx = Ax + Bx = 3.82 and C y = Ay + By = 2.76. The magnitude and direction of C are given by
2 C = Cx2 + C y = (3.82)2 + (2.76) 2 = 4.71

C = tan 1

Cy 1 2.76 = tan = 35.8 Cx 3.82

Assess: Using the method of vector components and their algebraic addition to find the resultant vector yields the same results as using the graphical addition of vectors.

3.25.

Solve:

Visualize: Refer to Figure P3.25 in your textbook. j j (a) We are given that A + B + C = 2i with A = 4i , and C = 2 . This means A + C = 4i 2 . Thus, B = ( A + B + C ) ( A + C ) = (2i ) (4i 2 ) = 6i + 2 . j j

(b) We have B = Bxi + By with Bx = 6 and By = 2. Hence, B = ( 6) 2 + (2) 2 = 6.3 j

= tan 1

By | Bx |

= tan 1

2 = 18 6

Since B has a negative x-component and a positive y-component, the angle made by B is with the x-axis and it is above the x-axis. Assess: Since | By | < | Bx |, < 45 as is obtained above.

3.26.

Visualize:

Solve:

(a) E = tan 1 1 = 45 1
2 F = tan 1 1 = 63.4

Thus = 180 E F = 71.6


(b) From the figure, E = 2 and F = 5. Using

G 2 = E 2 + F 2 2 EF cos = ( 2) 2 + ( 5) 2 2( 2)( 5)cos(180 71.6) G = 3.00. Furthermore, using sin sin(180 71.6) = = 45 2.975 5 Since E = 45 , the angle made by the vector G with the +x-axis is G = ( + E ) = 45 + 45 = 90. (c) We have
Ex = +1.0, Fx = 1.0, Gx = 0.0, G = and E y = +1.0 and Fy = +2.0 and G y = 3.0
2

( 0.0 )

+ ( 3.0 ) = 3.0,
2

and = tan 1

|G y | |Gx |

3.0 = tan 1 = 90 0.0

That is, the vector G makes an angle of 90 with the x-axis. Assess: The graphical solution and the vector solution give the same answer within the given significance of figures.

3.27.
Solve:

Visualize: Refer to Figure P3.27. From the rules of trigonometry, and

we

have

Ax = 4cos 40 = 3.1 and

Ay = 4sin 40 = 2.6. Also,


A + B + C = 0,

Bx = 2cos10 = 1.97

By = +2sin10 = 0.35.

Since

C = A B = ( A) + ( B) = (3.1i 2.6 ) + (+1.97i 0.35 ) = 1.1i 3.0 . j j j

3.28.

Visualize:

D u

Solve:

In the tilted coordinate system, the vectors A and B are expressed as:

A = (2sin15 m)i + (2cos15 m) j

and

B = (4cos15 m)i (4sin15 m) . j

j j Therefore, D = 2 A + B = (4 m)[(sin15 + cos15)i + (cos15 sin15) ] = (4.9 m)i + (2.9 m) . The magnitude of this vector is D = 5.7 m, and it makes an angle of = tan 1 (2.9 m/4.9 m) = 31 with the +x-axis. Assess: The resultant vector can be obtained graphically by using the rule of tail-to-tip addition.

3.29.

Visualize:

j j The magnitude of the unknown vector is 1 and its direction is along i + . Let A = i + as shown in the j diagram. That is, A = 1i + 1 and the x- and y-components of A are both unity. Since = tan 1 ( Ay / Ax ) = 45,
the unknown vector must make an angle of 45 with the +x-axis and have unit magnitude. Solve: Let the unknown vector be B = Bxi + By where j

Bx = B cos 45 =

1 B 2

and By = B sin 45 =

1 B 2

We want the magnitude of B to be 1, so we have


2 B = Bx2 + By = 1

1 1 B + B = 1 B2 = 1 B = 1 2 2 Hence, Bx = By = Finally, 1 1 B = Bxi + By = j i+ j 2 2 1 2

3.30.

Model: Carlos will be represented as a particle and the particle model will be used for motion under constant acceleration kinetic equations. Visualize:

Solve: Carlos runs at constant speed without changing direction. The total distance he travels is found from kinematics:

r1 = r0 + v0 t = 0 m + ( 5 m/s )( 600 s ) = 3000 m


This displacement is north of east, or = 25 from the +x-axis. Thus the position r1 becomes

r1 = (3000 m)(cos 25i + sin 25 ) = 2.7 km i + 1.27 km j j That is, Carlos ends up 1.27 km north of his starting position. Assess: The choice of our coordinate system is such that the x-component of the displacement is along the east and the y-component is along the north. The displacement of 3.0 km is reasonable for Carlos to run in 10 minutes if he is an athlete.

3.31. Visualize: The coordinate system (x,y,z) is shown here. While +x denotes east and +y denotes north, the +z-direction is vertically up. The vectors S morning (shortened as S m ), Safternoon (shortened as Sa ), and the total
displacement vector S total = Sa + S m are also shown.

j j Solve: S m = (2000i + 3000 + 200k ) m, and Sa = (1500i + 2000 300k ) m. The total displacement is the sum of the individual displacements. (a) The sum of the z-components of the afternoon and morning displacements is

Sza + Szm = 300 m + 200 m = 100 m, that is, 100 m lower.


(b) S total = Sa + S m = (500i + 5000 100k ) m, that is, (500 m east) + (5000 m north) (100 m vertical). The j magnitude of your total displacement is

S total =

( 500 )

+ ( 5000 ) + ( 100 ) m = 5.03 km


2 2

3.32.

Visualize:

Only the minute hand is shown in the figure. j j Solve: (a) We have S8:00 = (2.0 cm) and S8:20 = (2.0 cm)cos30i (2.0 cm)sin 30 . The displacement vector is

r = S8:20 S8:00 = (2.0 cm)[cos30i (sin 30 + 1) ] j = (2.0 cm)[0.87i 1.50 ] j j = (1.74 cm)i (3.00 cm)
(b) We have S8:00 = (2.0 cm) and S9:00 = (2.0 cm) . The displacement vector is r = S9:00 S8:00 = 0. j j Assess: The displacement vector in part (a) has a positive x-component and a negative y-component. The vector thus is to the right and points down, in quadrant IV. This is where the vector drawn from the tip of the 8:00 a.m. arm to the tip of the 8:20 a.m. arm will point.

3.33.

Visualize:

(a)

Note that +x is along the east and +y is along the north. j j Solve: (b) We are given A = (200 m) , and can use trigonometry to obtain B = (283 m)i (283 m) and + (173 m) . We want A + B + C + D = 0. This means C = (100 m)i j

D = A B C = (200 m ) + (283 m i + 283 m ) + (100 m i 173 m ) = 183 m i + 310 m j j j j The magnitude and direction of D are
D = (183 m) 2 + (310 m) 2 = 360 m and = tan 1 Dy Dx 310 m = tan 1 = 59.4 183 m

This means D = (360 m, 59.4 north of east). (c) The measured length of the vector D on the graph (with a ruler) is approximately 1.75 times the measured length of vector A . Since A = 200 m, this gives D = 1.75 200 m = 350 m. Similarly, the angle measured with the protractor is close to 60. These answers are in close agreement to part (b).

3.34.

Visualize:

(a) The figure shows Sparkys individual displacements and his net displacement.

Solve:

(b) Dnet = D1 + D2 + D3 , where individual displacements are

D1 = (50cos 45i + 50sin 45 ) m = (35.4i + 35.4 ) m j j D2 = 70i m D3 = 20 m j j Thus Sparkys displacement is Dnet = ( 35i + 15.4 ) m. (c) As a magnitude and angle,
Dnet = ( Dnet ) 2 + ( Dnet ) 2 = 38 m, x y

net = tan 1

( Dnet ) y = 24 | ( Dnet ) x |

Sparkys net displacement is 38 m in a direction 24 north of west.

3.35.

Visualize:

Solve: total

j We are given A = 5 m i and C = (1 m)k . Using trigonometry, B = (3cos 45 m)i (3sin 45 m) . The . The magnitude of r is displace-ment is r = A + B + C = (7.12 m)i (2.12 m) (1 m)k j

r = (7.12)2 + (2.12) 2 + (1)2 m = 7.5 m. Assess: A displacement of 7.5 m is a reasonable displacement.

3.36.

Visualize:

Solve: Assess:

We

have

v = vxi + v y j

= v||i + v j

j = v cos i + v sin .

Thus,

v|| = v cos = (100 m/s)cos30 = 86.6 m/s.

For the small angle of 30, the obtained value of 86.6 m/s for the horizontal component is reasonable.

3.37.

Visualize:

2.5 m/s (a) Since vx = v cos , we have 2.5 m/s = (3.0 m/s)cos = cos 1 = 34. 3.0 m/s (b) The vertical component is v y = v sin = (3.0 m/s) sin 34 = 1.7 m/s.

Solve:

3.38.

Visualize:

The coordinate system used here is tilted with x-axis along the slope. Solve: The component of the velocity parallel to the x-axis is v|| = v cos70 = v sin 20 = 10 m/s (0.34) = 3.4 m/s. This is the speed down the slope. The component of the velocity perpendicular v = v sin 70 = v cos 20 = 10 m/s (0.94) = 9.4 m/s. This is the speed toward the ground. Assess: A speed of approximately 10 m/s implies a fall time of approximately 1 second under free fall. Note that g = 9.8 m/s2. This time is reasonable for a drop of approximately 5 m, or 16 feet.

3.39.

Visualize:

Solve: (a) The river is 100 m wide. If Mary rows due north at a constant speed of 2.0 m/s, it will take her 50 s to row across. But while shes doing so, the current sweeps her boat sideways a distance 1 m/s 50 s = 50 m. Marys net displacement is the vector sum of the displacement due to her rowing plus the displacement due to the rivers current. She lands 50 m east of the point that was directly across the river from her when she started. (b) Marys net displacement is shown on the figure.

Visualize: Establish a coordinate system with origin at the tree and with the x-axis pointing east. Let A be a displacement vector directly from the tree to the treasure. Vector A is A = (100i + 500 ) paces. j This describes the displacement you would undergo by walking north 500 paces, then east 100 paces. Instead, you follow the road for 300 paces and undergo displacement

3.40.

B = (300sin 60i + 300cos60 ) paces = (260i + 150 ) paces j j

Solve: Now let C be the displacement vector from your position to the treasure. From the figure A = B + C. So the displacement you need to reach the treasure is C = A B = (160i + 350 ) paces. j

If is the angle measured between C and the y-axis,

= tan 1

160 = 24.6 350

2 You should head 24.6 west of north. You need to walk distance C = Cx2 + C y = 385 paces to get to the

treasure.

3.41. Visualize: A 3% grade rises 3 m for every 100 m horizontal distance. The angle of the ground is thus = tan 1 (3/100) = tan 1 (0.03) = 1.72.
Establish a tilted coordinate system with one axis parallel to the ground and the other axis perpendicular to the ground.

Solve: From the figure, the magnitude of the component vector of v perpendicular to the ground is v = v sin = 15.0 m/s.

But this is only the size. We also have to note that the direction of component is v = 15.0 m/s.

is down, so the

3.42.

Visualize:

Solve:

j The resulting velocity is given by v = vfly + vwind , where vwind = 6 m/s i and vfly = v sin i v cos .
6 and = 48.6. Thus the ducks should head 48.6 west of south. 8

Substituting the known values we get v = 8 m/s sin i 8 m/s cos + 6 m/s i . We need to have vx = 0. This j

means 0 = 8 m/s sin + 6 m/s, so sin =

3.43.

Model: Visualize:

The car is treated as a particle in this problem.

Solve:

(a) The tangential component is a|| = a sin 30 = (2.0 m/s 2 )(0.5) = 1.0 m/s 2 .

(b) The perpendicular component is a = a cos30 = (2.0 m/s 2 )(0.866) = 1.7 m/s 2 . Assess: Magnitudes of the tangential and perpendicular components of acceleration are reasonable.

3.44.

Model: Visualize:

We will treat the knot in the rope as a particle in static equilibrium.

Solve:

Expressing the vectors using unit vectors, we have F1 = 3.0i and F2 = 5.0sin 30i + 5.0cos30 . Since j F1 + F2 + F3 = 0, we can write F3 = F1 F2 = 0.5i 4.33 . The magnitude of F3 is given by j

F3 = (0.5) 2 + (4.33) 2 = 4.4 units. The angle F3 makes is = tan 1 (4.33/0.5) = 83 and is below the negative xaxis. Assess: The resultant vector has both components negative, and is therefore in quadrant III. Its magnitude and direction are reasonable. Note the minus sign that we have manually inserted with the force F2 .

3.45.

Visualize:

Use a tilted coordinate system such that x-axis is down the slope. j Solve: Expressing all three forces in terms of unit vectors, we have F1 = (3.0 N)i , F2 = + (6.0 N) , and F3 = (5.0 N)sin i (5.0 N)cos . j (a) The component of Fnet parallel to the floor is ( Fnet ) x = (3.0 N) + 0 N + (5.0 N)sin 30 = 0.50 N, or 0.50 N up the slope. (b) The component of Fnet perpendicular to the floor is ( Fnet ) y = 0 N + (6.0 N) (5.0 N)cos30 = 1.67 N. (c) The magnitude of Fnet is Fnet = ( Fnet ) x + ( Fnet ) y = (0.50 N) 2 + (1.67 N) 2 = 1.74 N. The angle Fnet makes

is

= tan 1

( Fnet ) y |( Fnet ) x |

1.67 N = tan 1 = 73 0.50 N

Fnet is 73 above the floor on the left side of F2 . 00000

3.46.

Visualize:

Solve:

Using trigonometry to calculate , we get = tan 1 (100 cm/141 cm) = 35.3. Expressing the three forces in unit j j j vectors, FB = (3.0 N)i , FC = (6.0 N) , and FD = + (2.0 N)cos35.3i (2.0 N)sin 35.3 = (1.63 N)i (1.16 N) . The total

j force is Fnet = FB + FC + FD = 1.37 N i 7.2 N . The magnitude of Fnet is Fnet =

(1.37 N) 2 + (7.2 N) 2 = 7.3 N.

net = tan 1
Fnet = (7.3 N,79 below x in quadrant III).

|( Fnet ) y |

7.2 N = tan 1 = 79 |( Fnet ) x | 1.37 N

3-1

Você também pode gostar